Đến nội dung

toanhoc2017 nội dung

Có 571 mục bởi toanhoc2017 (Tìm giới hạn từ 18-04-2020)



Sắp theo                Sắp xếp  

#728575 CHỌN ĐỘI TUYỂN HSG QUỐC GIA TỈNH HÒA BÌNH

Đã gửi bởi toanhoc2017 on 02-07-2021 - 22:58 trong Thi HSG cấp Tỉnh, Thành phố. Olympic 30-4. Đề thi và kiểm tra đội tuyển các cấp.

Bài bdt mình có cách làm như sau không dùng chebyshep.Áp dụng cauchyschawzt
$(\sum x^{2}(a+b+1))(\sum \frac{1}{a+b+1})\geq (x+y+z)^{2}$
và ta cần cm bdt $\sum \frac{1}{a+b+1}\leq 1$
ta có $a+b\geq \sqrt[3]{a^{2}b}+\sqrt[3]{ab^{2}}$ nên $\sum \frac{1}{a+b+1}\leq \sum \frac{1}{\sqrt[3]{a^{2}b}+\sqrt[3]{ab^{2}}+\sqrt[3]{abc}}=1$
nên bdt dc chứng minh

a=b=C=1



#728574 Đề chọn đội tuyển Quốc Gia Hà Tĩnh 2016-2017 (2 ngày)

Đã gửi bởi toanhoc2017 on 02-07-2021 - 22:57 trong Thi HSG cấp Tỉnh, Thành phố. Olympic 30-4. Đề thi và kiểm tra đội tuyển các cấp.

Ngày 1:
Câu 3:
k.png
a) Ta có: $A_1A_1' \perp BC \Rightarrow A_1A_1' \perp B_1'C_1'$
Chứng minh tương tự suy ra $A_1A_1',B_1B_1',C_1C_1'$ đồng quy tại $H'$ là trực tâm của tam giác $A_1'B_1'C_1'$
MẶt khác $H'A_1.H'A_1'=H'B_1.H'B_1'=H'C_1.H'C_1'$
Do đó $OH'$ là trục đẳng phương của $3$ đường tròn $(OA_1A_1'),(OB_1B_1'),,(OC_1C_1')$
Do đó $3$ đường tròn $(OA_1A_1'),(OB_1B_1'),,(OC_1C_1')$ cùng đi qua điểm $K$ khác $O$
b) Dễ thấy tam giác $A_1'B_1'C_1'$ là ảnh của tam giác $ABC$ qua phép đối xứng tâm $O$
Do đó $a,b,c$ cũng là độ dài $3$ cạnh của tam giác $A_1'B_1'C_1'$ và $OH=OH'$
Gọi $p'$ là chu vi của tam giác $A_{2}B_{2}C_{2}$ thì dễ thấy $p'=\dfrac{P}{2}$ ( Do tam giác $A_{2}B_2C_2$ là ảnh của tam giác $A_1'B_1'C_1'$ qua phép vị tự tâm $H$ tỉ số $\dfrac{1}{2}$
Do đó ta chỉ cần chứng minh $OK.OH'=\dfrac{abc}{2p'}$
Dễ thấy $OA_1' \perp B_2C_2$ nên $S_{OB_2A_1'C_2}=\dfrac{R.B_2C_2}{2}$ trong đó $R$ là bán kính của $(O)$
Chứng minh tương tự và cộng lại suy ra
$\dfrac{abc}{4R}=S_{A_1'B_1'C_1'}=\dfrac{R.p'}{2}\Rightarrow \dfrac{abc}{2p'}=R^2$
Lại có: $\widehat{OA_1A_1'}=\widehat{OA_1'A_1}=\widehat{OKA_1}$ nên $OA_1$ là tiếp tuyến của $(H'A_1K)$ suy ra $OK.OH'=OA_1^2=R^2$
Do đó ta có điều phải chứng minh
Câu 4: (Góp $1$ cách khác là đếm truy hồi)
Cho tập $S=\left\{1,2,3,...,n\right\}$. Gọi dãy $(a_1,...,a_n) $ là một dãy đẹp nếu có tính chất $2(a_1+a_2+a_3+...+a_k)\ \vdots\ k$ $\forall k=1,2,...,n$. Khi đó gọi $s_n$ là số dãy đẹp trong các hoán vị của $S$. Ta cần tính $s_{2016}$
Trước hết ta chứng minh $s_n=2s_{n-1}$ $\forall n$ chẵn
Thật vậy ta có: $2(\dfrac{n(n+1)}{2}-a_n) \vdots n-1$
Do đó $2a_{n}\equiv 2 (mod \ n-1)$ hay $a_n \equiv 1 (mod \ n-1)$
Do đó $a_n=1$ hoặc $a_n=n-1$
Rõ ràng nếu $(a_1,...,a_n) $ là một dãy đẹp và $a_1,...,a_n >1$ thì $(a_1-1,...,a_n-1) $ cũng là một dãy đẹp
Do đó nếu $n$ chẵn thì $s_n=2s_{n-1}$
Với $n$ lẻ và $n>3$. Đặt $n=2k+1 \Rightarrow k>1$
Xét tương tự trường hợp $n$ chẵn thì $a_n \equiv 1 (mod \ k)$
Do đó $a_n=1$ hoặc $a_n=n$ hoặc $a_n=k+1$
Tương tự trên $s_n=2s_{n-1}+t_n$ trong đó $t_n$ là số dãy đẹp có $a_n=k+1$
Giờ ta sẽ xét $a_{n-1}$
Ta có: $2\left(\dfrac{n(n+1)}{2}-a_{n-1}-(k+1)\right) \vdots n-2$
Do $n-2$ là số lẻ nên $a_{n-1}+k+1 \equiv 3 (mod \ n-2)$
Ta có: $a_{n-1}+k+1 \leq n+k+1< 2(n-2)+3$ (Với $k >1$)
Và $a_{n-1}+k+1>1+1+1=3$ nên suy ra $a_{n-1}+k+1 = (n-2)+3=n+1=2(k+1)$ nên $a_{n-1}=k+1$ suy ra vô lý
Do đó $t_n=0$ với $n$ lẻ và $n>3$
Kết hợp những điều trên suy ra $s_n=2s_{n-1}$ với mọi $n$ nguyên dương và $n>3$
Từ đây dễ dàng suy ra với mọi $n$ nguyên dương và $n>3$ thì $s_n=2^{n-3}s_3$
Mặt khác dễ thấy $s_3=6$ nên $s_n=2^{n-3}.6=3.2^{n-2}$ với mọi $n$ nguyên dương và $n>3$
Vậy $s_{2016}=3.2^{2014}$

iii



#728573 Đề thi Chọn Đội tuyển Dự thi HSG Quốc Gia Đà Nẵng 2014-2015

Đã gửi bởi toanhoc2017 on 02-07-2021 - 22:52 trong Thi HSG cấp Tỉnh, Thành phố. Olympic 30-4. Đề thi và kiểm tra đội tuyển các cấp.

a) Nhận xét: Khi thay dấu cộng thành trừ (và ngược lại), tính chẵn lẻ của tổng không đổi.
Với $n\equiv 1;2\left ( mod 4 \right )$ thì tổng $1+2+...+n$ lẻ
Vì vậy, $f\left ( n \right )=0$ khi $n\equiv 1;2\left ( mod 4 \right )$
b) Chứng minh $f\left ( n \right )\geq 2^{\frac{n}{2}-1}$
Với $n=3$ và $n=4$, mệnh đề trên hiển nhiên đúng.
Ta chứng minh rằng nến $n$ đúng thì $n+4$ cũng đúng
Xét dấu của các số $n+1;n+2;n+3;n+4$:
Nếu ta đặt các dấu là $\left ( n+1 \right )-\left ( n+2 \right )-\left ( n+3 \right )+\left ( n+4 \right )$ hoặc $-\left ( n+1 \right )+\left ( n+2 \right )+\left ( n+3 \right )-\left ( n+4 \right )$ và các dấu còn lại đặt như các bộ $f\left ( n \right )$ thì ta có bộ mới thỏa yêu cầu đề bài.
Đối với các bộ thuộc $f\left ( n \right )$ có $-1$ thì ta đổi thành$+1$ rồi thêm $\left ( n+1 \right )-\left ( n+2 \right )+\left ( n+3 \right )-\left ( n+4 \right )$ thì có được bộ mới
Đối với các bộ thuộc $f\left ( n \right )$ có $+1$ thì ta đổi thành$-1$ rồi thêm $-\left ( n+1 \right )+\left ( n+2 \right )-\left ( n+3 \right )+\left ( n+4 \right )$ thì có được bộ mới
Đối với các bộ thuộc $f\left ( n \right )$ có $-2$ thì ta đổi thành$+2$ rồi thêm $\left ( n+1 \right )+\left ( n+2 \right )-\left ( n+3 \right )-\left ( n+4 \right )$ thì có được bộ mới
Đối với các bộ thuộc $f\left ( n \right )$ có $+2$ thì ta đổi thành$-2$ rồi thêm $-\left ( n+1 \right )-\left ( n+2 \right )+\left ( n+3 \right )+\left ( n+4 \right )$ thì có được bộ mới
Vậy $f\left ( n+4 \right )\geq 4f\left ( n \right )\geq 2^{\frac{n+4}{2}-1}$
Suy ra đpcm
Chứng minh $f\left ( n \right )<2^n-2^{\left [ \frac{n}{2} \right ]+1}$
Gọi $g\left ( n \right )$ là số các bộ dấu sao cho tổng trên khác $0$
Ta cần chứng minh $g\left ( n \right )> 2^{\left [ \frac{n}{2} \right ]+1}$
Chứng minh bằng qui nạp
Với $n=3$ và $n=4$ thì ta có mệnh đề trên đúng.
Ta chứng minh với $n$ đúng thì $n+4$ cũng đúng.
Xét dấu của các số $n+1;n+2;n+3;n+4$
Đối với các bộ thuộc $f\left ( n \right )$, ta đặt dấu các số $n+1;n+2;n+3;n+4$ để tổng đại số chúng khác $0$. Có $14$ cách đặt dấu cho mỗi bộ như vậy.
Đối với các bộ thuộc $g\left ( n \right )$, ta đặt dấu các số $n+1;n+2;n+3;n+4$ để tổng đại số chúng bằng $0$. Có $2$ cách đặt như vậy cho mỗi bộ
Suy ra $g\left ( n+4 \right )\geq 14f\left ( n \right )+2g\left ( n \right )>14.2^{\frac{n}{2}-1}+2.2^{\left [ \frac{n}{2} \right ]+1}>2^{\left [ \frac{n+4}{2} \right ]+1}$
Vậy ta có đpcm

Bài hay



#728572 Đề thi chọn đội tuyển toán THPT chuyên Đại học Sư Phạm Hà Nội năm học...

Đã gửi bởi toanhoc2017 on 02-07-2021 - 22:46 trong Thi HSG cấp Tỉnh, Thành phố. Olympic 30-4. Đề thi và kiểm tra đội tuyển các cấp.

Câu BDT có cách khác như sau:
Không mất tính tổng quát,giả sử $x\geq y\geq z$
Nếu đưa $(x,y,z)\rightarrow (x-z,y-z,0)$ thì các hiệu $x-y,y-z,z-x$ cũng không đối , nhưng $x+y,y+z,z+x$ ban đầu sẽ giảm , suy ra VT giảm còn VP tăng.Từ đó ta đưa BĐT về chỉ cần xét với 3 biến là $(a,b,0)$ trong đó $a,b$ như trên. BĐT tương đương
$\frac{a+b}{(a-b)^{2}}+\frac{1}{a}+\frac{1}{b}\leq \frac{9}{a+b}$
Do BĐT thuần nhất nên ta có thể giả sử $a+b=2$, qua đó $b=2-a$, thay vào ta chỉ còn BĐT 1 biến,đặt $2x-x^{2}= t$ thì bdt tương đương
$9t^{2}+4\geq 12t$, từ đây ta tìm được $t$ và qua đó tìm được dấu bằng, $(x,y,z)\in \left ( (1+\frac{1}{\sqrt{3}})l,(1-\frac{1}{\sqrt{3}})l,0 \right )$


- Vừa phát hiện ra cách này khá giống cách của bạn @Daicagiangho1998-

Cách dài khó hiểu nhé



#728571 Đề thi chọn đội tuyển HSG QG trường THPT chuyên Đại Học Vinh năm học 2014-2015

Đã gửi bởi toanhoc2017 on 02-07-2021 - 22:43 trong Thi HSG cấp Tỉnh, Thành phố. Olympic 30-4. Đề thi và kiểm tra đội tuyển các cấp.

Bài 6. Cho $p$ là số nguyên tố thoả mãn $5^{(p-1)^2}-1$ không chia hết cho $p$. Tìm cặp số nguyên dương $(x,y)$ thoả mãn $p^x+5=y^p$.
Lời giải. Nếu $p=5$ thì $5^x+5=y^5$, ta suy ra $5|y$ nên $5^5|y^5$, mà $5^5 \nmid 5^x+5$, mâu thuẫn. Vậy $p \ne 5$.
Từ giả thiết ta dễ dàng suy ra $p \parallel 5^{p-1}-1$.
Dễ thấy rằng $p$ lẻ. Ta có $y^p \equiv y \equiv 5 \pmod{p}$. Đặt $y=pk+5$ với $k \in \mathbb{N}$. Ta có $$y^p-5= (pk+5)^{p}-5= (pk)^p+p \cdot (pk)^{p-1} \cdot 5+ \cdots + p \cdot (pk) \cdot 5^{p-1}+5 \left( 5^{p-1}-1 \right).$$
Để ý rằng $p^2| (pk)^p+ \cdots + p \cdot (pk) \cdot 5^{p-1}$ nhưng $p \parallel 5 \left( 5^{p-1}-1 \right)$ nên ta suy ra $p \parallel (pk+5)^p-5=y^p-5$. Do vậy $x=1$. Ta có $p+5=y^p$. Nhận thấy $y \ge 2$.
Với $y \ge 3$ thì chứng minh theo quy nạp $y^p \ge 3^p>p+5$ với mọi $p \ge 3$.
Với $y=2$ thì $p+5=2^p$ suy ra $3|p$, dẫn đến $p=3$. Thử lại thấy thoả mãn.
Vậy $\boxed{(x,y)=(1,3)}$.

Hay



#728570 ĐỀ CHỌN ĐỘI TUYỂN QUỐC GIA TỈNH THANH HOÁ 2014-2015

Đã gửi bởi toanhoc2017 on 02-07-2021 - 22:39 trong Thi HSG cấp Tỉnh, Thành phố. Olympic 30-4. Đề thi và kiểm tra đội tuyển các cấp.

Câu 2 ngày 2. Tìm tất cả các số nguyên $m \ge 2, n \ge 2$ thoả mãn $\frac{1+m^{3^n}+m^{2 \cdot 3^n}}{n}$ là số nguyên.
Lời giải. Để ý rằng cặp $(m,n)=(k,3)$ với $k \equiv 1 \pmod{3}, k \ge 2, k \in \mathbb{Z}$ luôn thoả mãn. Ta xét với $n \ge 4$.
Giả sử tồn tại số nguyên tố $p|n$ thoả mãn $\gcd (p,3)=1$. Ta có $n|1+m^{3^n}+m^{2 \cdot 3^n}$ nên $n|m^{3^{n+1}}-1$.
Đặt $l= \text{ord}_p(m)$ thì ta suy ra $l|3^{n+1}$ và $l|p-1$.
Nếu $l|3^n$ thì $p|m^{3^n}-1$ mà $p|1+m^{3^n}+m^{2 \cdot 3^n}$ nên $p|m^{2 \cdot 3^n}+2$. Ta lại có $p|m^{3^n}-1$ nên $p|m^{2 \cdot 3^n}-2 \cdot m^{3^n}+1= m^{2 \cdot 3^n}+2- 2 \cdot \left( m^{3^n}-1 \right) -3$. Từ đó dẫn đến $p|3$ hay $p=3$, mâu thuẫn. Vậy $l \nmid 3^n$. Do đó $l=3^{n+1}$. Khi đó $3^{n+1}|p-1$, mâu thuẫn.
Vậy $n=3^k$ với $k \in \mathbb{N}, k \ge 1$. Ta suy ra $m \equiv 1 \pmod 3$. Tuy nhiên ta lại có $$v_3 \left( m^{2 \cdot 3^n}+m^{3^n}+1 \right)= v_3 \left( m^{3^{n+1}}-1 \right)- v_3 \left(m^{3^n}-1 \right)=1.$$
Từ đây dẫn đến $n=3$, mâu thuẫn.
Vậy $(m,n)=(k,3)$ với $k \ge 2, k \equiv 1 \pmod 3, k \in \mathbb{Z}$. $\blacksquare$

Nghiệm cụ thể k có nhé



#728569 Chọn đội tuyển QG Tp Hải Phòng 2014-2015

Đã gửi bởi toanhoc2017 on 02-07-2021 - 22:37 trong Thi HSG cấp Tỉnh, Thành phố. Olympic 30-4. Đề thi và kiểm tra đội tuyển các cấp.

Bài hình ngày 1 thì quen rồi (nó trong dự tuyển IMO 199x gì đó,trong đề chọn đội tuyển tỉnh các năm trước,trong tài liệu chuyên toán hình học 10,.......)
Gọi $E,F$ là tiếp điểm của BC với $(O');(O'')$
Gọi $D'$ là điểm chính giữa cung $BC$ không chứa A.Dùng phép vị tự hoặc Talet ta được $M,E,D';N,F,D'$ đều thẳng hàng
$P_{D';(O')}=D'B.D'M=D'B^{2}=D'C^{2}=D'E.D'N=P_{D';(O'')}$$\Rightarrow$ D' thuộc trục đẳng phương của $(O'),(O'')\Rightarrow A,I,D'$ thẳng hàng (đpcm)
b,$DB^{2}=DI^{2}=DC^{2}=DE.DM\Rightarrow \bigtriangleup DIC$ cân ở D $\Rightarrow \angle DIC=\angle DCI\Rightarrow \frac{A}{2}+\angle ICA=\angle ICB+\frac{A}{2}\Rightarrow \angle ACI=\angle BCI\Rightarrow$ đpcm
Câu 3:Thay n bởi n+1 ta được $\frac{(n+2)^{3}}{3}a_{n+1}=(1.2a_{1}+2.3.a_{2}+...+n(n+1)a)+(n+1)(n+2)a_{n+1}$
Kết hợp với gt $a_{n}.\frac{(n+1)^{3}}{3}=1.2.a_{1}+2.3.a_{2}+...+n(n+1)a_{n}$ ta được $\frac{(n+2)^{3}}{3}a_{n+1}=\frac{(n+1)^{3}}{3}a_{n}+(n+1)(n+2)a_{n+1}\Leftrightarrow a_{n+1}=\frac{(n+1)^{3}}{n^{3}+5n^{2}+11n+10}a_{n}\Rightarrow a_{n+1}-a_{n}=\frac{-2n^{2}-8n-9}{n^{3}+5n^{2}+11n+10}a_{n}< 0\Rightarrow (a_{n})$ giảm và bị chặn dưới bởi 0 nên dãy hội tụ
\\Câu b $lima_{n}=0$ mới đúng chứ nhỉ?Chắc mình sai.........cần cao thủ trợ giúp??\\
Câu PTH Thay x=0 vào ta được $f(0+2f(y))=f(0)+\frac{y}{2}$.Dễ thấy $f$ đơn ánh (và cả toàn ánh nhưng không cần dùng)
Thay x=y=0$\Rightarrow f(2f(0))=f(0)\Rightarrow f(0)=0$.Khi đó $f(2f(y))=\frac{y}{2}$.
Thay $y=f(2y)$ (á phải dùng toàn ánh ) vào PTH ban đầu ta được $f(x+y)=f(x)+f(y)$ kết hợp với $f(x)\geq 0$ với mọi $x\geq 0\Rightarrow f$ cộng tính $\Rightarrow f(x)=x.$
Câu bất ngày 2 (korea MO 2014) Bằng kỹ thuật thay b=c;và chọn k max sao cho bất đẳng thức luôn đúng ta chọn được $b=\frac{1}{2}$
Thay $a=0;b=c=\frac{1}{2}\Rightarrow k\leq 4$
Với $k=4$ ta cần cm $\sum \frac{a}{1+9bc+4(b-c)^{2}}\geq \frac{1}{2}$
AD Cauchy Shwars ta có $VY=\sum \frac{a^{2}}{a+9abc+4a(b-c)^{2}}\geq \frac{(a+b+c)^{2}}{(a+b+c)+27abc-24abc+ab(a+b)+bc(b+c)+ac(a+c)}=\frac{1}{1+3abc+\sum 4ab(1-c)}=\frac{1}{1+4(ab+bc+ac)-9abc}\geq \frac{1}{2}\Leftrightarrow 9abc\geq 4(ab+bc+ac)-1$
Nhưng điều này luôn đúng do $abc\geq \prod (a+b-c)=\prod (1-2a)\Rightarrow 9abc\geq 4(ab+bc+ac)-1$

Dấu bằng khi nào



#728568 [Trường Xuân toán học miền nam 2016] Vietnam TST 2016 MOCK Test 2

Đã gửi bởi toanhoc2017 on 02-07-2021 - 22:24 trong Thi HSG cấp Tỉnh, Thành phố. Olympic 30-4. Đề thi và kiểm tra đội tuyển các cấp.

cho em xin hỏi kỹ thuật đánh giá ở trên có chuyên đề nào về nó không ạ

Kỹ thuật hay



#728567 ĐỀ THI OLYMPIC CHUYÊN KHOA HỌC TỰ NHIÊN NĂM 2015

Đã gửi bởi toanhoc2017 on 02-07-2021 - 22:12 trong Thi HSG cấp Tỉnh, Thành phố. Olympic 30-4. Đề thi và kiểm tra đội tuyển các cấp.

Xét $p=2$ => $x=5$

Xét $p\neq 2$
$3^p+4^p=x^2\Leftrightarrow (3+4)(3^{p-1}-3^{p-2}.4+...-3.4^{p-2}+4^{p-1})=x^{2}$
nên $x^{2}$ chia hết cho 7
Suy ra $(3^{p-1}-3^{p-2}.4+...-3.4^{p-2}+4^{p-1})$ chia hết cho 7
Mà $4\equiv -3(mod 7)$ nên $3^{p-1}-3^{p-2}.4+...-3.4^{p-2}+4^{p-1} \equiv 3^{p-1}-3^{p-2}.(-3)+...-3.(-3)^{p-2}+(-3)^{p-1}=p.3^{p-1}$ chia hết cho 7
Suy ra p=7. suy ra $n^{2}=3^{7}+4^{7}$( không tồn tại n thỏa mãn)

Vậy $p=2$

Chưa kỹ lắm



#728561 Đề chọn đội tuyển học sinh giỏi quốc gia tỉnh Quảng Ninh ngày 1 2016-2017

Đã gửi bởi toanhoc2017 on 02-07-2021 - 12:25 trong Thi HSG cấp Tỉnh, Thành phố. Olympic 30-4. Đề thi và kiểm tra đội tuyển các cấp.

Ta có đánh giá sau $a^2-ab+b^2 = \frac{1}{4}(a+b)^2 + \frac{3}{4} (a-b)^2 \geq \frac{1}{4}(a+b)^2 $
Do đó, ta có
$P \geq \sum \frac{a+b}{2\sqrt{ab} + 2 } $
Mặt khác ya có
$P-1 \geq \sum (\frac{a+b}{2\sqrt{ab}+2} -\frac{1}{3}) = \frac{(a+b-2\sqrt{ab} ) + 2(a+b-1)}{2\sqrt{ab}+2 } $
Ta chứng minh
$\sum \frac{a+b-1}{2\sqrt{ab}+2} \geq 0 $
Không mất tỉnh tổng quát, giả sử $a \geq b \geq c$
Khi đó thì $ab \geq ac \geq bc $
Do đó $\sum \frac{a+b-1}{2\sqrt{ab}+2} \geq \frac{2(a+b+c)-3}{2\sqrt{ab} +2} $
Mà ta có do $1=(a+b)(b+c)(c+a) \leq (\frac{2a+2b+2c}{3} )^3 => a+b+c \geq \frac{3}{2} $
Do đó $\sum \frac{a+b-1}{2\sqrt{ab}+2} \geq \frac{2(a+b+c)-3}{2\sqrt{ab} +2} \geq 0 $
Do đó $P \geq 1 $
Dấu bằng xảy ra khi $a=b=c=\frac{1}{2} $




#728560 Đề kiểm tra trường Đông toán học miền Nam 2015 - Bài kiểm tra số 1

Đã gửi bởi toanhoc2017 on 02-07-2021 - 12:08 trong Thi HSG cấp Tỉnh, Thành phố. Olympic 30-4. Đề thi và kiểm tra đội tuyển các cấp.

Bài 2 . Ta có bất đẳng thức tương đương $$\dfrac{\sum ab(c+b)^2(c+a)^2}{[(a+b)(b+c)(c+a)]^2}+\dfrac{5}{4}\geq \dfrac{6(ab+bc+ca)}{(a+b+c)^2}$$
Khai triển và nhóm trực tiếp, đồng thời đặt $\left\{\begin{matrix} p=a+b+c\\ q=ab+bc+ca\\ r=abc \end{matrix}\right.$
thì $\sum ab(c+b)^2(c+a)^2=abc\sum a^3+4abc\sum a.\sum ab+\sum a^2b^2.\sum ab=rp^3-pqr+q^3+3r^2$
Ta cần chứng minh
$\dfrac{rp^3-pqr+q^3+3r^2}{(pq-r)^2}+\dfrac{5}{4}\geq \dfrac{6q}{p^2}$
$\Leftrightarrow \dfrac{4rp^3+4q^3+17r^2+5p^2q^2-14pqr}{4(pq-r)^2}\geq \dfrac{6q}{p^2}$
$\Leftrightarrow 4p^5r+5p^4q^2-14p^3qr-20p^2q^3+17p^2r^2+48pq^2r-24qr^2\geq 0$
Chuẩn hóa $p=1$ thì ta cần chứng minh $r^2(17-24q)+r(48q^2-14q+4)-20q^3+5q^2\geq 0$
Với $q\leq \dfrac{1}{4}$ thì $r\geq 0$, suy ra $r^2(17-24q)+r(48q^2-14q+4)-20q^3+5q^2=r^2(17-24q)+r(48q^2-14q+4)+5q^2(1-4q)\geq 0$
Với $\dfrac{1}{4}\leq q\leq \dfrac{1}{3}$, áp dụng BĐT Schur ta có $r\geq \dfrac{4q-1}{9}$ thì $17-24q$ và $48q^2-14q+4$ đều dương
Khi đó ta cần chứng minh $\dfrac{(4q-1)^2}{81}(17-24q)+\dfrac{4q-1}{9}.(48q^2-14q+4)-20q^3+5q^2\geq 0\Leftrightarrow (1-3q)(4q-1)(23q+19)\geq 0$
Luôn đúng nên có điều cần chứng minh
Dấu bằng xảy ra khi $a=b=c$ hoặc $a=b;c=0$ và các hoán vị

Dùng cô si rất nhanh



#728556 Đề luyện tập olympic khối 10 VMF lần 2 tháng 7

Đã gửi bởi toanhoc2017 on 02-07-2021 - 09:54 trong Thi HSG cấp Tỉnh, Thành phố. Olympic 30-4. Đề thi và kiểm tra đội tuyển các cấp.

Một cách khác cho bài bất:
Ý tưởng cách này là sử dụng phương pháp p,q,r.
Bđt <=> $\frac{a^{2}+b^{2}+c^{2}+3(ab+bc+ca)}{(a+b)(b+c)(c+a)}+\frac{1}{8}(ab+bc+ca)\geq\frac{15}{8}$
Đặt ab+c=p, bc+ca+ab=q, abc=r
<=> $\frac{p^{2}+q}{pq-r}+\frac{1}{8}q\geq \frac{15}{8}$
<=> $8p^{2}+8q+pq^{2}-qr\geq 15pq-15r$
<=>$8p^{2}+8q+pq^{2}-qr-15pq+15r\geq 0$
<=>$72+8q+3q^{2}-qr-45q+15r\geq 0$ (p=3)
Ta sẽ cm bđt trên. Thật vậy, ta có một số bđt sau:
$\frac{p^{3}}{27}\geq r$
$r\geq \frac{p(4q-p^{2})}{9}$
=>$72+8q+3q^{2}-qr-45q+15r\geq 72+8q+3q^{2}-q.\frac{p^{3}}{27}-45q+15.\frac{p(4q-p^{2})}{9}=72+8q+3q^{2}-q.\frac{3^{3}}{27}-45q+15.\frac{3(4q-3^{2})}{9}=72+8q+3q^{2}-q-45q+20q-45=3q^{2}-18q+27=3(q-3)^{2}\geq 0$(luôn đúng)
=> Bđt được chứng minh. Dấu bằng xảy ra <=> a=b=c=1
=> Q.E.D

Mình nghi dung cô si hay



#728388 $\frac{bcx}{(x+y)(x+z)}+\frac{cay}{(y+z)(y+x)}+\frac{abz}...

Đã gửi bởi toanhoc2017 on 24-06-2021 - 22:41 trong Bất đẳng thức - Cực trị

Lời giải
Đặt $m=x+y,n=y+z,k=z+x$ thì $m,n,k$ là độ dài 3 cạnh của 1 tam giác
$$
\begin{array}{l}
3.6 \Leftrightarrow \dfrac{bc(m+k-n)}{mk}+\dfrac{ac(m+n-k)}{mn}+\dfrac{bc(n+k-m)}{nk}\leq \dfrac{(a+b+c)^2}{m+n+k}\\
\Leftrightarrow \dfrac{bc(m^2+k^2-n^2+2mk)}{mk}+\dfrac{ac(m^2+n^2-k^2+2mn)}{mn}+\dfrac{bc(n^2+k^2-m^2+2nk)}{nk}\leq (a+b+c)^2\\
\Leftrightarrow \dfrac{bc(m^2+k^2-n^2)}{mk}+\dfrac{ac(m^2+n^2-k^2)}{mn}+\dfrac{bc(n^2+k^2-m^2)}{nk}\leq a^2+b^2+c^2 \hfill \quad (*)
\end{array}
$$
Nhưng mặt khác do $m,n,k$ là độ dài 3 cạnh tam giác nên (*) có thể viết lại thành:
\begin{equation}
\;\;\;\;(3.1.1)
2bc\cos \widehat{N}+2ac\cos \widehat{K}+2ab\cos \widehat{M}\leq a^2+b^2+c^2
\end{equation}
Trong đó: $\widehat{M},\widehat{N},\widehat{P}$ là 3 góc của $\vartriangle MNP$ nhận $m,n,p$ thứ tự là cạnh đối diện tương ứng.
Và ${3.1.1}$ là bất đẳng thức lượng giác quen thuộc nên ta có đpcm.

Bai nay hay



#728248 Bai Hinh hay

Đã gửi bởi toanhoc2017 on 18-06-2021 - 19:24 trong Hình học

Làm ntn có đủ chặt chẽ không nhỉ.
Ta có $\angle AGE=\angle ADE=\angle ABC$ nên tứ giác EGMB nội tiếp. Suy ra AG . AM = AE . AB = AD . AC
Từ đó: $\angle BGC=\angle BGM+\angle CGM=\angle BEM+\angle CDM=\angle ABC+\angle ACB=\angle BHC$.
Suy ra 4 điểm B, H, G, C đồng viên.
Giả sử KH cắt đường tròn đường kính AH tại G' khác H.
Ta có KH . KG' = KE . KD = KB . KC nên G' là giao điểm của (BHC) và đường tròn đường kính AH.
Theo cách dựng ta có $G'\equiv G$.
Từ đó $\angle KGA=\angle HGA=90^o$ nên $KG\perp MG$.
Mặt khác đường nối tâm của (MBE) và (MCD) vuông góc với đường thẳng nối hai giao điểm của chúng nên ta có đpcm.

Cung hay nhung co cach cap 2 k pan



#728243 Bai Hinh hay

Đã gửi bởi toanhoc2017 on 18-06-2021 - 18:00 trong Hình học

Bạn nào giải cứu cân hình c gon xi nhé. mạng giai dài quá



#728242 Bai Hinh hay

Đã gửi bởi toanhoc2017 on 18-06-2021 - 17:58 trong Hình học

Cau c giai Sao
Trên mạng lời giải dài quá, k hay

Hình gửi kèm

  • de-thi-vao-lop-1-toan-da-nang-2021-result.jpg



#728224 $\frac{a}{b+c} + \frac{b}{c+d}+ \frac{c}{d + a}+\fra...

Đã gửi bởi toanhoc2017 on 17-06-2021 - 22:34 trong Bất đẳng thức và cực trị

Áp dụng bất đẳng thức Bunhiacopxki, ta có:
$VT\left[ {a\left( {b + c} \right) + b\left( {c + d} \right) + c\left( {d + a} \right) + d\left( {a + b} \right)} \right] \ge {\left( {a + b + c + d} \right)^2}$
Ta cần chứng minh:
$\begin{array}{l}
{\left( {a + b + c + d} \right)^2} \ge 2\left( {ab + bc + cd + da + 2ca + 2bd} \right)\\
\Leftrightarrow {a^2} + {b^2} + {c^2} + {d^2} \ge 2ca + 2bd\\
\Leftrightarrow {\left( {a - c} \right)^2} + {\left( {b - d} \right)^2} \ge 0
\end{array}$
Bất đẳng thức đã được chứng minh.

Hay



#728173 Đề thi chuyên KHTN vòng 2 năm 2021

Đã gửi bởi toanhoc2017 on 16-06-2021 - 11:35 trong Tài liệu - Đề thi

Goi ban nhe

Hình gửi kèm

  • FB_IMG_1623817894077.jpg



#728171 Đề thi chuyên KHTN vòng 2 năm 2021

Đã gửi bởi toanhoc2017 on 16-06-2021 - 11:29 trong Tài liệu - Đề thi

Bạn làm bài số và bđt như thế nào vậy :D

Bai bat dang thuc dung danh gia va bai so hoc dung dong du thuc la xong



#726812 Tìm GTNN của $P=\frac{x}{\sqrt{y+2021Z...

Đã gửi bởi toanhoc2017 on 10-05-2021 - 14:41 trong Bất đẳng thức và cực trị

Cho x,y,z là các số dương thỏa mãn điều kiện $x+y+z\geq 12$. Tìm GTNN của $P=\frac{x}{\sqrt{y+2021Z}}+\frac{y+2022}{\sqrt{z}}+\frac{z}{\sqrt{x+2023}}$




#725693 Đề thi HSG Toán 9 cấp thành phố Cần Thơ

Đã gửi bởi toanhoc2017 on 21-04-2021 - 07:37 trong Tài liệu - Đề thi

Điểm rơi đúng x=y=0,5 là bảo giải đúng .Dễ quá thì cũng nghi ngờ



#725230 $\sum \frac {ab}{1-ab}\leq \frac {3}{8}$

Đã gửi bởi toanhoc2017 on 11-04-2021 - 11:10 trong Bất đẳng thức - Cực trị

Ai có lời giải thuần bằng C-S hay AM-GM thì post lên nhé :)

Xét phép đổi biến $p,q,r$,ta sẽ có $r\in \left ( 0;\frac{1}{27} \right ]; q \in \left ( 0;\frac{1}{3} \right ]$.Theo Schur thì $r\geqslant \frac{4q-1}{9}$

BĐT cần chứng minh tương đương với:
$\sum \frac{ab}{1-ab}\leqslant \frac{3}{8}\Leftrightarrow \sum \frac{1}{1-ab}\leqslant \frac{27}{8}\Leftrightarrow \frac{\sum (1-ab)(1-bc)}{(1-ab)(1-bc)(1-ca)}\leqslant \frac{27}{8}$
$\Leftrightarrow \frac{3-2q+r}{1-q+r-r^{2}}\leqslant \frac{27}{8}\Leftrightarrow f(r)=27r^{2}-19r+11q-3\leqslant 0$

Dễ thấy $f'(r)=54r-19<0,\forall r \in \left ( 0;\frac{1}{27} \right ]$ nên $f(r)\leqslant f\left ( \frac{4q-1}{9} \right )=\frac{(16q+5)(3q-1)}{9}\leqslant 0,\forall q \in \left ( 0;\frac{1}{3} \right ]$

Ta có đpcm.

Hay



#725228 Vietnam TST 2021

Đã gửi bởi toanhoc2017 on 11-04-2021 - 10:59 trong Thi HSG Quốc gia và Quốc tế

Ngô Quý Đăng vẫn có tên trong danh sách dự thi Vietnam TST 2021 nhé

Cu Đăng tạch rùi ! Tiếc quá.Cu Lâm , Cu Nghĩa phong độ víp quá



#724422 Trại hè hùng vương 2019

Đã gửi bởi toanhoc2017 on 01-08-2019 - 21:20 trong Thi HSG cấp Tỉnh, Thành phố. Olympic 30-4. Đề thi và kiểm tra đội tuyển các cấp.

bạn nào gõ telex lưu lại chứ trôi mất




#724421 BÀI TS CHUYÊN V1 TUYÊN QUANG 2015

Đã gửi bởi toanhoc2017 on 01-08-2019 - 21:19 trong Hình học

mình nghĩ đề viết sai rồi